1993 AJHSME Problems/Problem 23

Revision as of 20:14, 19 August 2011 by Mrdavid445 (talk | contribs) (Created page with "==Problem== Five runners, <math>P</math>, <math>Q</math>, <math>R</math>, <math>S</math>, <math>T</math>, have a race, and <math>P</math> beats <math>Q</math>, <math>P</math> be...")
(diff) ← Older revision | Latest revision (diff) | Newer revision → (diff)

Problem

Five runners, $P$, $Q$, $R$, $S$, $T$, have a race, and $P$ beats $Q$, $P$ beats $R$, $Q$ beats $S$, and $T$ finishes after $P$ and before $Q$. Who could NOT have finished third in the race?

$\text{(A)}\ P\text{ and }Q \qquad \text{(B)}\ P\text{ and }R \qquad \text{(C)}\ P\text{ and }S \qquad \text{(D)}\ P\text{ and }T \qquad \text{(E)}\ P,S\text{ and }T$